Use the given recursion and starting value of to find :
Do the same for and :
(That's not a mistake. This just tells you that the 2nd and 3rd iterates are very close together and have at least the same first 5 digits after the decimal.)
Answer:
A
Step-by-step explanation:
Two facts need to guide your answer.
One
The highest power is odd: you know this because an even power would start on the left come down do it's squiggles if had any and wind up on the right going up.
This graph comes down on the left does it's squiggles and then goes further down on the right. That's the behavior of something whose highest power is odd.
Two
The leading coefficient, the number in front of the highest power must be minus. If it was positive as in y = x^3 the graph would be the mirror image of what it is.
Argument
B and D cannot be true. The highest power is even.
C is false because the leading coefficient is + 1.
So that leave A which is the answer.
The graph is included with this answer
Answer:
<h2>y = 3x - 3</h2>
Step-by-step explanation:
The slope-intercept form of an equation of a line:
<em>m</em><em> - slope</em>
<em>b</em><em> - y-intercept</em>
<em />
We have the slope <em>m = 3</em>, and the point <em>(2, 3)</em>.
Put the value of slope and the coordinates of the given pint (x = 2, y = 3) to the equation of a line:
<em>subtract 3 from both sides</em>
Finally:
<h2>Solution (a) :-</h2>
Let the cost of the less expensive comic book be x .
Then :-
Using this equation , we can find the cost of the less expensive comic book .
Price of the less expensive book :-
Cost of the less expensive book is 107 .
Therefore , the epice of the less expensive book = $ 107 .
<h2>Solution (b) :-</h2>
Cost of the more expensive book = 11x
Which means :-
Therefore , the price of the more expensive book = $ 1177 .
Answer:
40.8
Step-by-step explanation:
When you divide 350 by 51 you get 6.8. Multiply that by 6 you get 40.8